La ricerca ha trovato 1608 risultati

da edriv
23 set 2009, 17:04
Forum: Fisica
Argomento: Campo elettrico di un piano
Risposte: 6
Visite : 7081

Tanto per dimostrare che ho capito qualcosa, rispondo a julio. L'argomento di Nonno Bassotto si applica anche al caso alpha generico, però NON porta allo stesso risultato. Invece che disporre le cariche su un piano, disponiamole su un reticolo \mathbb Z \times \mathbb Z , che approssima bene il pian...
da edriv
23 set 2009, 16:51
Forum: Fisica
Argomento: Campo elettrico di un piano
Risposte: 6
Visite : 7081

Ah ok, la spiegazione di Kirill mi ha aiutato particolarmente.

Grazie a tutti comunque.
da edriv
23 set 2009, 16:22
Forum: Fisica
Argomento: Campo elettrico di un piano
Risposte: 6
Visite : 7081

Campo elettrico di un piano

Scusate, io ho un libro di fisica che mi dice che il campo elettrico di un piano con carica uniforme ha la stessa intensità ovunque. Non riesco a capire questo: cioè, se io allontano una particella dal piano, visto che la distanza da ogni punto del piano aumenta, non dovrebbe diminuire la forza di a...
da edriv
21 set 2009, 17:23
Forum: Discorsi da birreria
Argomento: Decimillesimo utente
Risposte: 45
Visite : 16280

ma anche no... (da parte mia :P)
da edriv
17 set 2009, 18:49
Forum: Teoria dei Numeri
Argomento: Intervalli senza primi
Risposte: 5
Visite : 1923

Il punto b) non mi convince troppo... non c'era un teorema che diceva che le uniche potenze consecutive sono 8 e 9?
da edriv
15 set 2009, 14:24
Forum: Olimpiadi della matematica
Argomento: IMO 2009
Risposte: 75
Visite : 35716

ahah fantastica :D
da edriv
14 set 2009, 12:02
Forum: Matematica non elementare
Argomento: Definizione di valore atteso
Risposte: 3
Visite : 2355

Definizione di valore atteso

Mi chiedevo...
In probabilità esiste una definizione di valore atteso non del tipo:
"il valore atteso è ciò che ricavi facendo questo integrale"
ma più carina e meno costruttiva, del tipo:
"il valore atteso è una cosa che soddisfa le seguenti proprietà:
- bla
- bla
- bla bla bla"
da edriv
05 set 2009, 15:18
Forum: Olimpiadi della matematica
Argomento: Video senior 2008
Risposte: 1
Visite : 2037

Video senior 2008

Qualcuno sa se sono da qualche parte i video delle lezioni dell'ultimo stage senior?
da edriv
26 ago 2009, 10:07
Forum: Matematica non elementare
Argomento: Se i punti sono troppi staranno stretti!
Risposte: 3
Visite : 2535

Ancora più easy: consideriamo le palle di centro razionale (in ogni coordinata) e raggio razionale, che sono numerabili. Ad ogni punto isolato associamo una di queste palle che contenga il punto e sia disgiunta dagli altri punti. Così otteniamo una funzione iniettiva dall'insieme che vogliamo contar...
da edriv
24 ago 2009, 09:35
Forum: Teoria dei Numeri
Argomento: p|1^k+2^k+...+p^k
Risposte: 8
Visite : 2677

Comunque faccio notare che la sua soluzione usa i generatori
da edriv
22 ago 2009, 14:29
Forum: Matematica non elementare
Argomento: SNS 2000/2001 n 3
Risposte: 33
Visite : 13935

A questo punto è molto interessante fare lo stesso problema togliendo gli estremi dal segmento, e il perimetro dal triangolo!
da edriv
21 ago 2009, 19:44
Forum: Matematica non elementare
Argomento: SNS 2000/2001 n 3
Risposte: 33
Visite : 13935

Tanto per chiarezza: -sbagliare è umano -credo che lo scopo di questo forum "sovversivo" non sia prendere in giro chi ha sbagliato ma correggerlo ed eventualmente cercare di fargli capire dove sta l'errore, anche perchè questi commenti ironici non portano a nulla Leggi un po' troppo tra l...
da edriv
10 ago 2009, 18:53
Forum: Matematica non elementare
Argomento: SNS 2000/2001 n 3
Risposte: 33
Visite : 13935

allora mi ero preso un abbaglio...non tanto per la surgettività quanto per come avevo inteso il problema. Io pensavo che la funzione riguardasse le coordinate dei punti e non i punti stessi, cioè pensavo che come nel piano cartesiano f(x) fosse la coordinata y del punto :oops: Ora ho capito grazie ...
da edriv
08 ago 2009, 20:43
Forum: Matematica non elementare
Argomento: \phi e uno strano limite
Risposte: 8
Visite : 4955

Comunque direi che il limite fa:

$ \displaystyle \frac{\pi^2}{3} - \lim_{n \rightarrow \infty} \frac{1}{n^2} $

non so se si può scrivere in maniera più compatta...
da edriv
06 ago 2009, 12:43
Forum: Geometria
Argomento: Cresce il poligono, cresce il perimetro
Risposte: 3
Visite : 2415

Cresce il poligono, cresce il perimetro

Se due figure convesse del piano sono contenute l'una nell'altra, la più grande ha perimetro maggiore. Carino eh? :D Figure è un termine un po' generico... dimostratelo per i poligoni che è praticamente equivalente. Di questo problema c'è una bella soluzione giusto oltre il limite dell'"element...